Fórum de Matemática
DÚVIDAS? Nós respondemos!

Um Fórum em Português dedicado à Matemática
Data/Hora: 18 abr 2024, 11:17

Os Horários são TMG [ DST ]




Fazer Nova Pergunta Responder a este Tópico  [ 3 mensagens ] 
Autor Mensagem
MensagemEnviado: 26 ago 2014, 03:56 
Offline

Registado: 26 ago 2014, 03:44
Mensagens: 4
Localização: São Paulo
Agradeceu: 3 vezes
Foi agradecido: 0 vez(es)
Como calcular o limite de:

\(lim_{ (x,y)\rightarrow (0,0)} \frac{x^{2}y^{2}}{\left ( x^{2}+y^{2} \right )^{\frac{3}{2}}}\)


Topo
 Perfil  
 
MensagemEnviado: 26 ago 2014, 10:53 
Offline

Registado: 21 jan 2011, 11:31
Mensagens: 947
Localização: Portugal
Agradeceu: 11 vezes
Foi agradecido: 126 vezes
Normalmente estudam-se os limites direcionais para ver se não tem limite. Neste caso, vão todos dar um valor: 0.
Por exemplo, na direção (1,0),

\(\lim_{x\to 0, y=0}\frac{x^2y^2}{(x^2+y^2)^{\frac{3}{2}}}=\lim_{x\to 0}\frac{x^2.0}{(x^2)^{\frac{3}{2}}}=0\)

Uma vez tendo um candidato a limite, que pode ser confirmado noutras direções, provamos que 0 é mesmo o limite. Assim, vemos o limite "de quando a norma do vetor (x,y) tende para 0" (isto é, quando me aproximo do ponto (0,0), seja em que trajetória for) do módulo da função menos 0 (repara que se tendemos para zero, esse módulo tenderá para zero por valores positivos ou zero).

\(\lim_{||(x,y)|| \to 0} \left| \frac{x^2y^2}{(x^2+y^2)^{\frac{3}{2}}}-0 \right|\)

Se eu conseguir majorar a expressão do módulo por funções mais simples, tal que no fim consigo calcular o limite e é zero, então a expressão inicial tenderá para 0 também.

Isto é, se \(\lim_{||(x,y)|| \to 0} 0 \le \lim_{||(x,y)|| \to 0} \left| \frac{x^2y^2}{(x^2+y^2)^{\frac{3}{2}}}-0 \right| \le ... \le \lim_{||(x,y)|| \to 0} A(x,y)\)

e \(\lim_{||(x,y)|| \to 0} A(x,y)=0\), então posso concluir que o meu limite inicial tende também para 0.

Neste caso, temos

\(\lim_{||(x,y)|| \to 0} \left| \frac{x^2y^2}{(x^2+y^2)^{\frac{3}{2}}}-0 \right| =\)
\(\lim_{||(x,y)|| \to 0} \frac{|x|^2|y|^2}{(x^2+y^2)^{3}} \le\)
\(\lim_{||(x,y)|| \to 0} \frac{||(x,y)||^2||(x,y)||^2}{||(x,y)||^{3}}\)

já que \(|x| \le ||(x,y)||\) e o mesmo para \(|y|\)

Assim,

\(\lim_{||(x,y)|| \to 0} \frac{||(x,y)||^2||(x,y)||^2}{||(x,y)||^{3}} =\)
\(\lim_{||(x,y)|| \to 0} \frac{||(x,y)||^4}{||(x,y)||^{3}} =\)
\(\lim_{||(x,y)|| \to 0} ||(x,y)|| = 0\)

Assim, provamos que o limite é 0.

_________________
José Sousa
se gostou da resposta, divulgue o fórumdematemática.org

O Binômio de Newton é tão belo como a Vênus de Milo.
O que há é pouca gente para dar por isso.

óóóó---óóóóóó óóó---óóóóóóó óóóóóóóó
(O vento lá fora.)

Álvaro de Campos, 15-1-1928


Topo
 Perfil  
 
MensagemEnviado: 26 ago 2014, 13:40 
Offline

Registado: 21 jul 2013, 00:22
Mensagens: 673
Localização: Manchester
Agradeceu: 93 vezes
Foi agradecido: 340 vezes
Pode usar Coordenadas Polares :

\(x=rcos\theta \;\;\;\;\;\; y=rsen\theta \;\;\;\; (x,y) \to 0 \;\;\;\; , \;\;\;\; r \to 0\)



\(\lim_{r \to 0} \; \frac{r^4cos^{2} \theta sen^{2}\theta}{(r^2)^{\frac{3}{2}}}\)


\(\lim_{r \to 0} \; \frac{r^4cos^{2} \theta sen^{2}\theta}{r^{3}}\)


\(\lim_{r \to 0} \; r cos^{2} \theta sen^{2}\theta=0\)


Topo
 Perfil  
 
Mostrar mensagens anteriores:  Ordenar por  
Fazer Nova Pergunta Responder a este Tópico  [ 3 mensagens ] 

Os Horários são TMG [ DST ]


Quem está ligado:

Utilizadores a ver este Fórum: Nenhum utilizador registado e 25 visitantes


Criar perguntas: Proibído
Responder a perguntas: Proibído
Editar Mensagens: Proibído
Apagar Mensagens: Proibído
Enviar anexos: Proibído

Pesquisar por:
Ir para:  
cron